0 Daumen
246 Aufrufe

Ich habe folgende Folge gegeben und soll zeigen, dass deren Grenzwert 1 ist.

\( a_{n}=\left(1-\frac{1}{n^{2}}\right)^{n} \)

Ich soll zu vorgegebenem ε > 0 ein n0 ∈ ℕ bestimmten, dass für alle n ≥ n0 gilt [an - 1] < ε.

Ich habe das Prinzip denke ich verstanden, jedenfalls sollte man im Beispiel nach n auflösen.

Das gleiche wollte ich hier auch machen, allerdings weiß ich nicht wie ich mit dem n im Exponenten umgehen soll.

Avatar von

1 Antwort

0 Daumen

|  ( 1 - 1 / n^2)^n - 1 | kleiner eps

Die Klammer könntest du mit dem binomischen Lehrsatz auflösen, das

gäbe dann sowas wie

|  1  -  n * (1/n^2) +  xxxxxxxxx - xxxxxxxxxx + xxxxxxxxx ....      - 1 | kleiner eps

Die Teile mit dem xxxxxxx brauchst du nicht, denn der Betrag von

-  n * (1/n^2) +  xxxxxxxxx - xxxxxxxxxx + xxxxxxxxx .

ist kleiner als  n*(1/n^2)  also brauchst du nur 

n*(1/n^2) < eps

1/n   <  eps

n > 1/eps.

Avatar von 288 k 🚀

hmm

könnte ich nicht einfach sagen, da 1/n^2 gegen 0 läuft und dann 1/n^2 quasi wegfallen lassen.

Ich hätte dann allerdings am Ende 0 < ε stehen. Und was mir das sagen soll, wüsste ich auch nicht

Ein anderes Problem?

Stell deine Frage

Willkommen bei der Mathelounge! Stell deine Frage einfach und kostenlos

x
Made by a lovely community